Anda di halaman 1dari 56

(D)

87.Is X>Y ?

1). X+Y>0

2). Y^X<0

ANS : (C)

Stmt 1) X+Y>0. doesn’t give us any information about values of X and Y. We can have X=1 and Y=0
or X=0 and Y=1. INSUFF

Stmt 2) Y^X<0. Y^x can be <0 only when y is negative. X could be positive (odd numbers) or –ve
(odd) for the inequality to hold. INSUFF

Given X+Y>0 and Y^X<0 => Y is negative and X has to be positive (so that the sum becomes
positive). Hence X>Y. SUFF

(C)

88. An=An-1 +5, A5=31. A1=?

ANS: 11

A5 = A4+5 = A3+10 = A2+15 = A1+20

31 = A1+20 => A1=11

89. Which is more economic, copying or buying a $12.5 book with K pages?

1). K> 85

2). Copying a book with K+15 pages will cost more than $14

ANS: (E)

Stmt (1) K>85. Doesn’t give any idea about the cost of photocopy per page. INSUFF

Stmt (2) Copying a book with K+15 pages will cost more than $14. Let c be the cost of photocopy
per page. Given c(k+15)>14. We can have any combination of c and K such that they will satisfy
this inequality and also Kc will be greater than or less than 12.5. INSUFF

Combining (1) and (2) lets take K=85 => c(K+85)>14 => c(85+15)>14 => c>.14.

Since c>.14 and K>85, we have Kc>85*.14 => Kc>11.9. Hence Kc could be less than or greater
than 12.5. So we cannot say what would be more economical. INSUFF

(E)

90. Tom has x books and he can completely place them in the bookshelf by placing 10 copies in
each line. If 10 more books are added in, he can completely place them in the bookshelf by placing
12 copies in each line. X=?
1). x<96

2). x>24

ANS: (A)

Given x books can be completely places with 10 copies in each line => x=10p.

Given x+10 books can be completely placed with 12 copies in each line => x+10 = 12q => x =
12q-10

=> 10p = 12q-10 => 10(p+1) = 12q

Hence we are looking for a multiple of 10 and 12 here which would be 60K (since 60 is the LCM of
12 and 10).

Stmt 1) x<96. Only one multiple of 10 and 12 possible which is 60. => 10(p+1)=60 => p=5.
Hence x=50. SUFF

Stmt 2) x>24. There are infinite values possible. INSUFF

(A)

91. A and P took three tests. A's score in the first and second test is 10 and 4 higher than B's,
respectively. If P's average score for the three tests is 3 higher than that of A, in the third test, P's
score is how much higher than that of A?

ANS: 23

Lets A’s scores be A1, A2 and A3 and B’s score be B1, B2 and B3.

Given A1 = B1+10, A2 = B2+4 and (B1+B2+B3)/3 = 3 + (A1+A2+A3)/3

Take the third equation:

(B1+B2+B3)/3 = 3 + (A1+A2+A3)/3 => B1+B2+B3 = 9 + A1+A2+A3 (now substitute first 2


eqns):

=> B1+B2+B3 = 9 + B1+10 + B2+4 + A3

=> B3 = A3+23 => In third test, B’s score is 23 more than that of A.

92. A, B, C, and D took a certain test, if the sum of their scores is 400, is score of B greater than
that of C?

1). A+B=100+C+D

2). A+C=80+B+D

ANS: (C)

Given A+B+C+D=400
Stmt 1) A+B=100+C+D => Substituting in above equation gives C+D=150 and A+B=250. Which
essentially gives the information that C<=150 and B<=250 (scores cant be negative). Nothing else
can be said. INSUFF

Stmt 2) A+C=80+B+D => Substituting, gives B+D=160 and A+C=240. Again, nothing else can be
said. INSUFF

Combining (1) and (2), (A+B) + (B+D) – (C+D) – (A+C) = 250+160-150-240 => 2B-2C=20 => B-
C=10 => B>C. SUFF

93. What is the remainder when 43^43+ 33^33 is divided by 10?

ANS: 0

Units digit for power of any number with 3 in the units place follows pattern 3,9,7,1.

Units digit for 43^43 = 7 (remainder of 43 divided by 3 is 1. So the units digit will be 3)

Units digit for 33^33 = 7

Units digit of (43^43 + 33^33) = Units digit for 43^33 + Units digit for 33^33 = 0 (discard the 1).

Units digit when it is divided by 10 is same as the units digit of the number itself = 0.

94. Sequence A consists of 10 consecutive odd numbers and B consists of 5 consecutive even
numbers. If the least number in A is 7 greater than the least number in B, the average of the
numbers in A is how much greater than average of numbers in B?

ANS : 12

Let least number of B=x => least number in A = x+7

Sequence B = x, x+2, x+4, x+6, x+8 (consecutive odd) => average = [5x+20]/5 = x+4

Sequence A = x+7, x+9, x+11, x+13, x+15, x+17, x+19, x+21, x+23, x+25 (consecutive
even)=> average = x+18

Average of A – Average of B = 12

95. If r, s, and t are integer, is r^3 *s*t^4 negative?

1) r*t is negative

2) s is negative

ANS: (E)

The sign of r^3 *s*t^4 basically depends upon the sign of r and s. Since t is raised to an even
power, it will not contribute to the overall sign of the term.

Stmt 1) r*t is negative => either r is negative or t is negative, but both are not negative => If r is
negative and t is positive -> r^3 *s*t^4 will be negative, OR if r is positive and t is negative, r^3
*s*t^4 will be positive. Inconsistent values. INSUFF
Stmt 2) s is negative. Nothing is known about r. The term depends upon signs of both r and s.
INSUFF

Combining (1) and (2), the sign of r cannot be determined exactly and hence sign of the term r^3
*s*t^4 cannot be determined. INSUFF

(E)

96. S=1/32 + 1/33 + ... +1/64, S is in which of the following range?

0.5<S<1

ANS: 0.5<S<1

The question is best done with upper bound and lower bound. There are 32 terms in the sequence.

Upper bound for S will be when all the terms of the sequence are 1/32 (largest value). S(max) =
32*1/32 = 1

Lower bound for S will be when all the terms of the sequence are 1/64 (smallest value). S(min) =
32*1/64 = 0.5

Range = S(min)<S<S(max) = 0.5<S<1

97. One tank has a capacity of 64800 gallon, one gallon water weights 8.3 kg. A pile can fill the
tank with a constant speed in12 hours, what is the weight of the water filled into the tank per min?

ANS: 64800*8.3/(12*60)

A pile fills the tank (64800 gallon) in 12 hrs => the pile’s filling rate 64800/12 gallon/hr =
64800/(12*60) gallon/min

Weight of the water filled per minute = rate of pille (per min) * weight of water = 64800 *
8.3/(12*60) kg/min

98. If mn>0, whether m^3n^2>0?

1). m>0

2). n>0

Given mn>0 => either (m>0 and n>0) OR (m<0 and n<0)

ANS: (D)

Stmt 1) m>0 => n>0 => m^3n^2>0. SUFF

Stmt 2) n>0 => m>0 => m^3n^2>0. SUFF

(D)

99. Is x^2+y^2 divisible by 5?

1). When x-y is divided by 5, the remainder is 1


2). When x+y is divided by 5, the remainder is 3

ANS: (C)

Stmt 1) When x-y is divided by 5, the remainder is 1 -> nothing can be said about x^2+y^2.
INSUFF

Stmt 2) When x+y is divided by 5, the remainder is 3 -> nothing can be said about x^2+y^2.
INSUFF

Combining (1) and (2). From stmt 1 x-y = 5k+1 and stmt 2 x+y=5p+3. Square both eqn and add
them

2(x^2+y^2) = 25(k^2+p^2)+2*5*(k+2p)+10. On dividing both sides by 2, the first term is


25*(k^2+p^2)/2 which may or may not be divisible by 2 depending on k and p. However,
x^2+y^2 has to be integral value since x and y are integers => 25*(k^2+p^2)/2 should also be
an integer with 25 as a factor => x^2+y^2 is divisible by 5. SUFF

(C)

100. R and S are integers, is R/S a finite decimal?

1). R is the factor of 100

2). S is the factor of 100

ANS: (B)

Stmt 1) R is the factor of 100 -> a decimal being finite depends on the denominator. INSUFF

Stmt 2) S is the factor of 100 -> all the factors of 100 are multiples of 2 or 5. When 2 or 5 are
denominators, decimal is always finite. SUFF

(B)

101. A and B are integers, is A*B even?

1). A+B is odd

2). A is even.

ANS: (D)

Stmt 1) A+B is odd => either A is odd or B is odd but both are not odd => atleast one of them is
even. So AB is nothing but (odd*even) which will always be even. SUFF

Stmt 2) A is even. If A is even, AB will always be even. SUFF

(D)

102. If 56<x<66, where x is an integer. x=?


1). When x is divided by 2, the remainder is 1
2). x+1 is divisible by 3.
ANS: (E)

Stmt 1) When x is divided by 2, the remainder is 1 -> x can be any odd number between 56-66.
INSUFF

Stmt 2) x+1 is divisible by 3 -> x could be 59, 62, 65. INSUFF

Combining (1) and (2) we get that x can be 59 or 65. INSUFF

(E)

103. The figure above shows the shape of a mirror. The mirror has a semicircle in each end of a
rectangle. If the rectangle is 9 long and the ratio of the area of the rectangle to area of two
semicircles is 9 to pi, what is the width of the rectangle?

ANS: 4

Breadth of the rectangle = 2*r (r is radius of the circle)

Ratio of the areas (rectangle/circle)= 9*2r/pi*r^2 = 18/pi*r = 9/pi (given) => r=2.

Width = 2*r = 4.

104. A rope consists of 40% of A and other matters. A certain time later, 50% of A were lost and
the other remained. Now, the rope consists what percent of A?

ANS: 25%

Let the matter be 100%. A is 40% of matter = 40. 50% of A has lost => 20 of A remains in the
rope. But the rope has also reduced by 20.

So final percent of A in rope = 20/80 *100= 25%

105. If X/Y=3/5, and X+600/Y+500=4/5, X+600 is how much less than Y+500?

ANS: 300

5(X+600)=4(Y+500) => 5x+3000=4y+2000

And 5x=3y => 3y+3000=4y+2000 => y=1000 => x=600.

X+600 = 1200

Y+500 = 1500

=> +600 is 300 less than Y+500

106. Which of the following is terminating decimal?


I.1/12
II.1/10^2
III.1/2^10

ANS: II and III


I: Since 1/12 has 3 in the denominator, it will be a non- terminating decimal.

II: Since 1/100 =0.01 -> terminating

III: 1/2^10 -> since there are only 2’s in the denominator, the decimal is definitely terminating.

107. When a positive integer is divided by 4, the remainder is r; when divided by 9, the remainder
is R. What is the greatest possible value of r^2+R? [Uncertain]
23, 21, 17, 13, 11

ANS: 17

The greatest remainder when 4 is the divisor = 3 = r

The greatest remainder when 9 is the divisor = 8 = R

Greatest value of r^2+R = 9+8=17.

108. 35 percent of a number is decreased by 15percent, and the result is 25 percent of the original
number. What is the value of the number?
Author's answer is 1.5
The question seems incorrect.

ANS: Sorry!

109. 2x^2+5x=12, what is the difference between the bigger root and the smaller root?

ANS: 5.5

2x^2+5x-12=0

X = [-5 +/- root(25+4*12*2)]/ 4 = [-5 +/- root(121)]/4 = -16/4 or 6/4 = -4 or 1.5

Difference = 1.5+4 = 5.5

Number Number of
of Times people
0 3
1 2
2 10
3 3
4 5
5 3
110. The table above shows number of people corresponding to the number of the times. What is
the median of the number of times that at least is one?
ANS: 2
We have to find median number of times.
The Times series looks like
00011222222222233344444555
Clearly 2 is the median.
111. Is number 3 the tens?digit of x?
1). When x is divided by 100, the remainder is 30
2). When x is divided by 110, the remainder is 30
ANS: (A)
Stmt 1) When x is divided by 100, the remainder is 30 -> Since the remainder is 30, the units and
tens digit of the number are obviously 0 and 3 respectively. SUFF
Stmt 2) When x is divided by 110, the remainder is 30 -> x could be 250, 140, 1130 or anything.
No specific 100s digit. INSUFF
(A)
112. How many prime factors does x have?
1). X is factor of 7200
2). 180 is factor of x
ANS:(C)
Stmt 1) X is factor of 7200 -> Since we don’t know which factor of 7200 is X. We cant say anything
about its prime factors. INSUFF
Stmt 2) 180 is factor of x -> x could have prime factors other than that of 180. INSUFF
Combining (1) and (2) x is a factor of 7200 and 180 is a factor of x => x has only 2,3
and 5 as prime factors. SUFF
(C)
113. x/3 is between 3 and 100, and it is the square of a prime number. How many such x are
possible?
ANS: 4 numbers
x/3 = k^2 (where k is prime). x=3*k^2.
3<k^2<100 => root(3)<k<10. Given k is prime, k could be 2,3,5 and 7. So 4 values of x are
possible 12, 27, 75, 147
114. How many numbers between 1 and 100 are not divisible by 2 and 3?
30 31 32 33 34
ANS: 33
100 – number div by 2 – number div by 3 + number div by 2&3 (6) = 100-50-33+16 = 33.
I think there is some concern over “inclusive” or not. I have considered inclusive, but if you have
got the concept correct its all right.
115. The height median of the 5 children in family A is 118 cm. The boy in family B is 128cm. How
many children in A are taller than the boy in B?
1). The average height of children in A is 120cm.
2). The second height in A is 130cm.
ANS: (B)
Stmt 1) The average height of children in A is 120cm -> Cant say anything about how many
children in A are taller than B. INSUFF
Stmt 2) The second height in A is 130cm -> That means second tallest child in A is 130. The tallest
would be obviously greater than 130cm. Both of them will be taller than boy in family B. rest of the
children in A are smaller. SUFF
(B)

116. If r and s are root of x^2+bx+c=0, rs<0?

1). b<0

2). c<0

ANS: (B)

Stmt 1) b<0, r+s=-b. Cant say anything about product of the roots. SUFF

Stmt 2) c<0 -> rs<0 (product of the roots of a quadratic of the form ax^2+bx+c=0 is c/a). SUFF

117. (x+2)(x+3)/(x-2)>0, x is integer. If x<5, how many x are possible?

ANS: 3 values

For positive values of x, only those >= 2 will satisfy the equation. x=2,3,4.
For negative values, no values satisfy.

[I agree there is an issue with value 2. Since at x=2, the whole thing goes to infinity. However, in
this case since numerator is positive, the value will go to +ve side, hence I am considering that as
positive.

However, the approach to this question is pretty simple, there is no point getting stuck at this
question. I am sure, the question in actual GMAT test would be worded in better way so as to allay
any confusions]

118. Set I is defined such that, 1). if x is in the set, -x also is in the set. 2). if x and y are in the
set, then xy is in the set. Is 12 in the set?

1). 2 is in the set

2). -3 is in the set

ANS: (C)

Stmt 1) 2 is in the set -> Just gives us the info that 2 is in the set => -2 is in the set => 2*-2 is
also in the set (condition 2) and so on …. But we don’t have knowledge of any factors of 3, hence it
cannot be determined whether 12 will be in the set. INSUFF

Stmt 2) Same reasoning as above -> INSUFF

Combining (1) and (2) 2 is in set and -3 is in the set => -6 is in the set => 6 is in the set (cond 1)
=> 12 is in the set (apply cond (2) on 6 and 2). SUFF

(C)

119. How many sides does a polygon have?

1). All the sides have the same length

2). The sum of all interior angles is 1440

ANS:(B)

Stmt 1) All the sides have the same length -> All we can see here is that it is a regular polygon.
Could have any number of sides. INSUFF

Stmt 2) The sum of all interior angles is 1440 -> (2n-4)90 = 1440 => n=10. SUFF

(B)

120. x is between 90/245 and 3/4.Which of the following could be x?

1/2; some fraction equal to 0.743; some fraction coming to 0.367

ANS: 1/2

90/245 and 3/4. Now 90/245<90/180 => 90/245<1/2. Ofcorse 1/2<3/4 => 90/245<1/2<3/4
121. Mary goes from a to b and then from b to c. Speed for a to b is 40 feet/min and speed for b to
c is 60feet /min. She took 3 more hrs to go from a to b and the distance between a and b was 90
miles more. What was the time taken for b to c?

1. she took less than 45 mins

2. she took more than 42 mins.

ANS: T2=385 hrs

B to C : d1, v1 = 40 ft/min = 40*0.000189393939*60 miles/hr = 0.45 miles/hr

C to D : d2, v2 = 60 ft/min = 60*0.000189393939*60 miles/hr = 0.68 miles/hr

d1=d2+90 (given) => v1t1 = v2t2+90 => 0.45t1 = 0.68t2 + 90

t2+3=t1 (given)

0.45(t2+3) = 0.68t2 + 90

=> 0.45t2 + 1.35 = 0.68t2 + 90

t2 = (90-1.35)/(0.23) = 385 hrs

I am not sure what the two choices are doing there. I am sure something wrong with the question
itself. The concept seems straightforward.

122. A disc contains 10 songs. How many tracks do the songs occupy?

1). each song averagely occupies …tracks

2). Median is ...

ANS: (A)

Stmt 1) each song on an average occupy x tracks. So 10 songs will occupy 10x tracks. SUFF

Stmt 2) Cant say. INSUFF

(A)

123. The number of rooms of hotel G is 10 less than twice of that of H, and two hotels totally have
425 rooms. How many rooms does G have?

ANS: 280 rooms

G = 2*H-10 and G+H=425 => 2H-10+H=425 => 3H=435 => H=145 => G=280

124. 34/67<x< 151/200, x could be which of the following numbers?

I. 1/2

II. 35/67
III. 150/201

ANS: II and III only

A very good question. I am giving you a shortcut of the theorem I have used. I am leaving the proof
of the theorem to you.

Theorem: (a+x)/(b+y) is greater than a/b if and only if (bx-ay)>0. (Vice Versa is also
true)

I. x=1/2. We know that 34/68 < 34/67 (increasing just the denominator decreases the fraction) =>
1/2 < 34/67 and hence it doesn’t lie in the given interval. FALSE

II. x=35/67. we can easily see that 35/67 will be greater than 34/67 (compare the numerators
when denominators are same). So 34/67 < 35/67. [please note that just knowing that 35/67 is
greater than 34/67, doesn’t help] We HAVE to check whether 35/67 < 151/200 also ( otherwise it
will lie outside the given range and in GMAT they can easily trick us into making this mistake of not
checking with 151/200).

151/200 = (35+116)/(67+133)

So as per theorem, (35+116)/(67+133) > 35/67 iff 67*116 – 35*133 > 0. Which is true (you can
do quick multiplication or reason it out. Here is a quick method:

Is (35+32)(133-15) – 35*133 >0 ?

=> Is 35*133 – 15*35 + 32*133 – 15*32 – 35*133 >0?

=> Is 32*133 – 15 * (35+32) > 0

=> Is 32*133 – 15*67>0

We know 32 > 15 and 133 >67 => 32*133 > 15*67. So the above condition is true.

Hence 35/67 < 151/200.

So 34/67 < 35/67 < 151/200. TRUE

III x=150/201. ofcorse 150/201 < 151/200 (decreasing the numerator and increasing the
denominator both have decreasing effect on the fraction).

To find whether 34/67 < 150/201 use the theorem in same way as above. TRUE

Hence II and III only

125. Integers x and y have three digits and z is the sum of x and y. Is the tens?digit of z equal to
the sum of the tens?digits of x and y?

1). Both x and y have units?digits greater than 6

2). The sum of tens?digit of x and y is 7

ANS (A)
Stmt 1) Both x and y have units?digits greater than 6 -> There will be a “1” carried over while
adding the ten’s digit => At the tens digit of z we will always get 1+ sum of the tens digit (of x and
y). Hence the answer to the asked question is always NO. SUFF

Stmt 2) The sum of tens?digit of x and y is 7 -> Cant say, if there is a 1 carried over from the unit’s
digit then the ten’s digit of z will be 8 otherwise it will remain 7. INSUFF

(A)

126. Sequence a1, a2, a3, …an, after the first term, each term is equal to the previous term minus
the following term. a7=?

1). a5-a8=?/P>

2). a6=a8+?/P>

ANS: SORRY!

127. Is x+3y even?

1). x-3y is odd

2). x-y is odd

ANS: (D)

Stmt 1) x-3y is odd -> either x is odd or 3y is odd but not both. This condition also implies x+3y
will be odd (either x is odd or 3y is odd). SUFF

Stmt 2) x-y is odd. Either x is odd or y is odd.

If x is odd, y has to be even, hence 3y will also be even => x+3y is odd+even -> odd

If x is even, y has to be odd, hence 3y will also be odd => x+3y is odd+even -> odd.

No inconsistencies. SUFF

(D)

128. A person makes a sit-up excises plan. First day, he will do five sit-up, then 6 in the second
day, 7 in the third day…How many sit-up will he do in the following 16 days?

ANS: 200

Simple AP. S = 5+6+7 …… 21

a1=5, n=16 and d=1

Sum of AP = Sn = n/2 * ((2a+(n-1)d) = 200

129. Point (r,s) lies on the line L. Is the line’s intercept with axis-x greater than r?

1). L has a negative slope.


2). s<0

ANS: (C)

Let the equation of line be y=mx+c. The x intercept is x=-c/m. Since (r,s) lies on this line, it also
satisfies the equation of the line => s=rm+c => r = (s-c)/m

To find r>x we should have (s-c)/m – (-c/m) >0 => whether (s-c+c)/m >0 => whether s/m >0?

Stmt 1) L has a negative slope => m is negative, but we need information about s as well. INSUFF

Stmt 2) s<0 -> But we need information about m as well. INSUFF

Combine (1) and (2) -> both m and s are negative => s/m>0 => r>x. SUFF

130. X=10^n*25^2, what is the unit’s digit of x?

1). Forget…useless

2). n^2=1

ANS: (E)

Note: Since nothing is given about n, n could be positive or negative. This is very important to get
this question correct.

Stmt 1) – Nothing . INSUFF

Stmt 2) n^2 =1. n could be +/-1. if n=1 units digit is 0, if n=-1 units digit is 2. INSUFF

Cant really combine (1) and (2). INSUFF

(E)

131. 1/root2-root50=?

ANS : -9/2*root2

The question actually reads (1/root2) – root50 = (1/root2) – 5*root2 = (1-10)/root 2 = -9/root2 =
-(9*root2) /2 (multiply numerator and denominator by root 2).

132. If x is a prime number, is 3xy even?

1). x>2

2). y is odd

ANS : (C)

Stmt 1) x>2. Cant say anything because y could be even or odd. INSUFF

Stmt 2) y is odd -> cant say anything because x could be 2 (even prime number) or any other
prime number. INSUFF
Combine (1) and (2), x>2 and y is odd => x has to be odd (2 is the only even prime) and y is odd
=> 3xy has to be odd (definitely not even). SUFF

133. If b is an integer and b<10, x=1+ b/100. b=?

1). 1<=b<=3

2). The thousandths?digit of 10X^2 is equal to the tens?digit of x^2

ANS: (E)

Stmt 1) Cant say anything about unique value of b. INSUFF

Stmt 2) The thousandths digit of 10X^2 is equal to the tens digit of x^2 -> Since b<10

10x^2 = 10 ( 1+ 2b/100 + b^2/10000) = 10 + b/5 + b^2/1000 =>

x^2 = 1 + 2*b/100 + (b^2)/10000 => 1 + 20b/1000 + b^2/1000.

Lets concentrate on x^2. If b>0, it is evident (since b<0 given) that the units place of x^2 will be 1
and tens place of x^2 will be 0. Now check the b^2/1000 term in the 10x^2 => that for digits 1-9,
there square (units digit of square) will figure at the thousandth place. So thousandth place is
determined by units digit of square of integers 1 – 9. None of them can have “0” and hence there is
no possible value of b (0<b<10) which satisfies the condition.

b=0, x^2 = 1 (tens digit 0), 10x^2 = 10.000 (thousandth digit 0). Satisfies.

However, this is not the only value of b which satisfies. Take b=-10, -20, -30 etc each of them will
satisfy the given condition. [I took the hint from the fact that they gave b<10, I am always curios to
test the conditions on the limits, to see if they give some insight]. INSUFF

Combining (1) and (2) doesn’t really give us any value of b. INSUFF

(E) for me.

134. Of the 24 positive integers, all have the units?digit of 5, 1/3 have tens?digit of 0, 1/3 have
tens?digit of 1, 1/3 of tens?digit of 2. What is the tens?digit of sum of 24 numbers?

ANS: 6

The sum of the units digit = 5*24 = 120. So “12” is carried over.

Sum of ten’s digit (with the carry)= 8*0+8*1+8*2 + 12 = 36. Of which “3” will carry over. So the
ten’s digit will be 6.

135. The revenue of a certain company increased x percent from 1997 to 1998, and decreased x
percent from 1998 to 1999. If the revenue decreased 9% from 1997 to 1999, x=?

ANS: x=30%

Let the revenue be R in 1997

=> Revenue in 1998 = R’ = R*(1+x) (taking x in percentage)


=> revenue in 1999 = R’(1-x/100) = R*(1+x)(1-x)

Given revenue decrease from 1997 to 1999 is 9% => R(1-.09) = R(1+x)(1-x)

=> 1-0.09 = 1-x^2 => x^2 = 0.09 => x=0.3 => x=30%

136. Six years ago, a person deposited a certain amount of money in account A at 3% simple
annual interest rate, and other amount in account B at 4% simple annual interest rate, respectively.
If there was no any withdraw or deposit during the six years, is the current interest in A greater
than that in B?

1). The amount invested in A was $1000 more than that in B

2). The investment in account A obtained an interest of $10 last year.

ANS (C)

SI = p*r*t/100

Stmt 1) The amount invested in A was $1000 more than that in B -> But the difference in Simple
interest is dependent upon the actual amount invested (Principal). INSUFF

Stmt 2) The investment in account A obtained an interest of $10 last year -> This will give us the
principal for A. But still no information on investment in B. INSUFF

Combining (1) and (2) we can get principal for both A and B. rate and time are already given, the
given question can be answered. SUFF

(C)

137. The ratio of the number of registered publican to the number of registered democrats is 3/5. If
600 Publican and 500 Democrats were added in, the ratio became 4/5. What is the difference
between the two current numbers?

ANS: 300

p/d = 3/5 => 5p = 3d

(p+600)/(d+500) = 4/5 => 5p+3000 = 4d + 2000 => 3d+3000=4d+2000 => d=1000

p=1000*3/5 = 600

=> Current numbers => p+600 = 1200 and d+500 = 1500

=> difference = 1500-1200 = 300

138. Set S and T have 100 numbers, respectively. Is standard deviation of S greater than or less
than that of T?

1). Range of S is greater that of T

2). Average of S is greater than that of T

ANS: (E)
Stmt 1) Range of S is greater that of T -> Range can be greater still standard deviation can be less
(imagine a large value noted as a spike in the distribution while others are very close to the
average)). INSUFF

Stmt 2) Average of S is greater than that of T -> Average can be greater, still the standard deviation
can be less (or 0) if all values in S are equal to average. INSUFF

Combine (1) and (2) Still cant say anything. Imagine this distribution

S : 0 0 0 3 6 6 6 range = 6 , AV = 3 , SD = 1/7 * root(9+9+9+9+9+9) = 1/7 root(54)

T : 0 4 4 4 4 4 8 range = 8, AV = 4 , SD = 1/7 * root(16+0+0+0+0+0+16) = 1/7 root(32)

INSUFF

(E)

Basically, SD is more about distribution around the mean, and it hardly depends on the range as
such. This can also be seen from the formula

SD^2 + mean^2 = (Sum of squares of values)/n -> [clearly not dependent on range]

139. In a certain test, how many of the 35 students in a certain class got a score less than 85?

1). The average score of all students is 85

2). The median score of all students is 85

ANS: (E)

Stmt (1) The average score of all students is 85 -> Cant say anything. All students could have 85
and the average will still be 85. INSUFF

Stmt (2) The median score of all students is 85 -> Still cant say anything. All students could still
have 85 and the median would be 85. Or some can be less and some can be greater than 85 (with
85 as middle value). INSUFF

Combining (1) and (2) we still do not get any information. INSUFF

(E)

140. Is the tens?digit of x greater than that of y?

1). x-y=37

2). The units?digit of x is ?greater than that of y

ANS: (E)

Stmt 1) x-y=37, could be 109-72 or 99-62. INSUFF

Stmt 2) The units?digit of x is ?greater than that of y. Still could be 109-72 or 99-62. INSUFF

Combining (1) and (2) also we don’t get consistent answer. INSUFF
(E)

141. X+Y=?

1). x/y=2

2). y^2=4

ANS: (E)

Stmt 1) x/y =2. x and y could have any values since nothing is given about them. INSUFF

Stmt 2) y^2=4 => y= +/- 2. No information about x. INSUFF

Combining (1) and (2) y = +/-2. For y=2, X+Y=6 and for y=-2, X+Y = -6. Inconsistent. INSUFF

(E)

143. A company has 10 male and 15 female employees. If 15 people drive to office and 10 people,
including 8 female take bus, how many male drive to office?

ANS: 8 males

10 people including 8 females take bus => 2 males take bus => 10-2=8 males drive.

144. A car has a 15 gallon-tank. With one gallon gasoline, the car can travel 30 miles when driving
in the city, 20miles in the countryside. What expression is the fraction of the full fuel tank of a car
uses when driving 60 miles in city and 30 miles in the country?

ANS: (60/30+30/20) /15

Average = total miles / total gallons = (60/30 + 30/20) / 15

145. If P is a prime number greater than 2, which of the following could be a prime number?

2p, p^2, p+2, p/2, (p-1)/p

ANS: p+2

Note: Only integers can be categorized as prime

2p: Obviosly even with more than 2 factors (p,2,1,2p)

p^2: square of prime is not prime

p+2: could be prime (example 5+2)

p/2: decimal – Not qualified

p-1/p: some fraction <1 Not qualified

146. If 300<X<400, is the tens?digit of x greater than 5?


1). The units?digit of x is greater than 4

2). When x is added with 237, the hundreds?digit will be equal to 6

ANS : (B)

Stmt 1) The units?digit of x is greater than 4 -> Cant say. Could be 344 or 364. INSUFF

Stmt 2) When x is added with 237, the hundreds?digit will be equal to 6 -> Minimum such x would
be 600-237 = 363. Hence the minimum value of x is 363 => 10s digit for x is greater than 5. SUFF

(B)

147. An equipment cost $10,000. After the first year, 80% of the value of the previous year value
will be remained. 9 years later, how much will be the equipment value, in thousand dollars?

ANS: 10(0.8)^9

After every year 80% of the value remains.

If present price is p.:

After 1 year = 0.8p

After 2 yrs = 0.8*0.8*p = p*0.8^2

After 3 yrs = p*0.8^3

So after 9 years = p*0.8^9.

Since p is 10 thousand. The price after 9 years in thousands = 10*0.8^9

148. A, B, and C are the sides of a triangle. If A=4, B=3, which of the following can express the
value of C?

ANS: 1<C<7

(difference of the sides)< third side < (sum of the two sides)

4-3 < C < 4+3 => 1<C<7

149. x<y<z, where x, y, and z are positive integer, which of the following must greater than
(y^x)(y^z)?

ANS: Sorry!

150. K is a positive integer. If the sum of integers from -15 to K, inclusive is 51, K=?

ANS: 18

I think question is asking sum of consecutive integers.

S= 51 = n/2(-30+(n-1)) => 102 = -31n + n^2 => n^2-31n-102 = 0 => n=34 (discard the
negative value). K=-15+33 = 18.
K=18.

151. A square brick with area of 9 sq. inches is 1/8 inches thick. A number of bricks are spread in a
line, and continues a length of 144 inches. If the bricks were stacked one by one, what is the
height?

ANS: 6 inches

Total bricks: 144/3 = 48

height: 1/8 inches * 48 bricks= 6 inches

152. Which of lines R and L has a greater intercept with axis-y?

1). R has a greater intercept with axis-x than L.

2). R is parallel line L

ANS : (E)

Stmt 1) R has a greater intercept with axis-x than L. Based on the slope, R could have less or
greater intercept with axis-y. INSUFF

Stmt 2) R is parallel to line L -> R may have higher intercept or lower intercept. INSUFF

Combine (1) and (2) let R: y=mx+c1 and L: y=mx+c2. [slopes are same]

From (1), -m/c1 > -m/c2 (x intercepts can be found by putting y=0 in the equations).

We need to find if c1>c2?

Lets assume m is positive: -m/c1 > -m/c2 => -1/c1 > -1/c2 => 1/c2 > 1/c1.

Lets assume m is negative: -m/c1 > -m/c2 => -1/c2 > -1/c1 => 1/c1 > 1/c2.

As you can see the actual relation finally depends upon the signs of m, c1 and c2. INSUFF

(E)

153. Tax amount = taxable income * tax rate (not mentioned that the rate is fixed)=(total income-
some deductions)* tax rate.

A person’s taxable income = total income - some deductions, and one of the deduction is donation.
If his donation resulted in a $217 tax decreasing, how much did he donate?

1). the tax rate for the portion of taxable income that is over than 7500 is 28%

2). His taxable income is 7753

ANS: (C)

Let the actual income of the person be x. The donated amount be d. Remainder deductions be p.

The actual taxable income of the person = x-p-d (taking all deductions)
The taxable income of the person without the donation = x-p

Stmt 1) the tax rate for the portion of taxable income that is over than 7500 is 28% -> Cant
say anything, we have no idea if the actual income or the taxable income. INSUFF

Stmt 2) His taxable income is 7753 -> K = (x-p-d) is given. Nothing can be done without the tax
rate. INSUFF

Combine (1) and (2) K (taxable income) =7753. since K=x-p-d => x-p = K+d. However we just
know that the 28% rate is applied to the portion of the income above 7500. So let r be the rate
applied to the portion below 7500.

So total tax = 7500*r + 253*0.28 + 217 = 7500*r + (253+d)*0.28

=> 217 = d*0.28

=> d = 217/0.28 = 775. SUFF

(C) [Thanks Topo for poiniting the mistake out]

154. A person’s income consists of wage and deduction. This month, his wage is 180% of the
deduction. The deduction is what percent of the total income?

ANS: 35.6%

T=w+d

w = 1.8d

T = 2.8d

d = 1/2.8 * T = 0.36T => d = 35.6% of total income

155. A hiker walking at a constant rate of 4 miles per hour is passed by a cyclist traveling in the
same direction along the same path at a constant rate of 20 miles per hour. The cyclist stops to wait
for the hiker 5 minutes after passing her, while the hiker continue to walk at her constant rate. How
many minutes must the cyclist wait until the hiker catches up?

ANS: 20 mins

Distance between cyclist and hiker after 5 minutes (when cyclist stops) = 5/60 * (20-4) = 4/3 miles

Time taken by the hiker to cover this distance = 4/3 / 4 hr = 1/3 hr = 20 minutes

156. Is line A’s slope less than B’s?

1). A and B intersect at point P (10,1)

2). the y-intercept of A is greater than that of B.

ANS: (C)

Let equations of A: y=m1x+c1 and B:= y=m2x+c2


Stmt 1) A and B intersect at point P (10,1) -> Infinite lines intersect at (10,1). A and B could be any
of those lines. We need more information to determine the relation. INSUFF

Stmt 2) the y-intercept of A is greater than that of B -> Since the slopes are not decided, it could be
anything.

Combining (1) and (2) A and B pass through (10,1) => 1=10m1+c1 and 1=10m2+c2

Subtracting the equations 0 = 10(m1-m2)+(c1-c2) => (c1-c2)=-10(m1-m2).

From (2) its given c1>c2 or c1-c2>0 => -10(m1-m2)>0 => m2-m1>0 => m2>m1. (Gives Yes to
the asked question. SUFF

(C)

157. Students in the cafe, each either like tea or dislike tea, each either like coffee or dislike coffee.
If 2/3 students dislike tea, and in those students dislike tea, 3/4 dislike coffee. How many students
like coffee?

1). Total number of the students is 120

2). 40 students like tea

ANS: (E)

Stmt 1) Total number of the students is 120 -> 80 don’t like T. 40 Like T.

Of 80 who don’t like T, 3/4 dislike like coffee = 60 dislike coffee => 20 like coffee. However we are
not sure how many of those who like T dislike coffee [so the distribution of other 40 is not known].
Hence nothing can be said. INSUFF

Stmt 2) 40 students like T -> same as (1) condition. INSUFF

Combining (1) and (2) we don’t get any extra useful information. INSUFF

(E)

158. A company get a 50000 dollar contract of a project, the project only cost labor and material, is
the profit larger than 15000?

ANS: (C)

1).Labor cost is twice the cost of material

2).Profit is larger than Labor cost

Stmt 1) Labor cost is twice the cost of material -> 2M+M = 50000 => M=50000/3 => L = 2*
50000/3 = 33333.333. Nothing is known about Profit. INSUFF

Stmt 2) Profit is larger than Labor cost -> But nothing is know about the labor cost. INSUFF

Combine (1) and (2) -> Profit is greater than L => P > 33333.333. Hence P > 15000. SUFF

(C)
159. What is the unit's digit of X?
1). x/(10^n)=25^2
2). n^2=1

ANS: (E)

Stmt 2) x/(10^n)=25^2 => x = 10^n * 25^2. since nothing is known about n (could be negative),
units digit cannot be determined. INSUFF

Stmt 2) n^2 =1. No relation of n to x is given. INSUFF

Combine (1) and (2), n could be +/-1. for n=1, units digit =0, for n=-1, units digit = 2.
Inconsistent. INSUFF

(E)

160. Both B and W's wage are greater than 50,000. Is W>B?
1). B is closer to 50,000 than W
2). The difference between B and 35,000 is less than the difference between B and W

ANS: (D)

Stmt 1) B is closer to 50,000 than W. Already given B,W>50000. Hence B<W. SUFF

Stmt 2) The difference between B and 35,000 is less than the difference between B and W => B-
35000 < B-W => Since B is greater than 5000, 0<B-35000<B-W => B-W>0 => B>W. SUFF

(D)

161. x and y are 2-digt integers. What is the difference between two tens' digit?
1). x-y=27
2). Units' digit of x minus the units' digit of y is greater than 3

ANS: (E)

Stmt 1) x-y=27. Could be any set of 2 digit numbers: 99,72 or 68, 41 or 58,31 or 93,66. INSUFF

Stmt 2) Units' digit of x minus the units' digit of y is greater than 3 -> Nothing else is given. The
ten’s digits could be anything. INSUFF

The units digit of x is greater than y => If the integers are positive, No carry over required from the
tens digit during the subtraction => the difference in ten’s digit is fixed = 2. [he he! not so fast!]
However, since it is not given that the numbers have to be positive, the two digit numbers could be
negative also (x=17, y=-10). So inconsistent values. INSUFF

(E)

162. Each of ten items was cost C dollars. If 8 items were sold out at price of D (d>c) and 2 items
were sold out at price of 1.1C, what is the ratio of profit to cost?
1).d=c+8
2).d=1.2c

ANS (B)

Total Profit = 8*(d-c) + 2*0.1c


Stmt 1) d=c+8 -> d-c = 8. P = 64+.2c. Ratio cannot be found. Hence INSUFF

Stmt 2) d=1.2c -> P = 1.6c+0.2c => P=1.8c. Ration can be found. SUFF

163. Is X^3>y^2
1). y=x^2
2). x>x^2

ANS: (C)

Stmt 1) y=x^2 -> The inequality become Is x^3>x^4. Since nothing is know about x, INSUFF

Stmt 2) x>x^2. No relation is given between x and y. INSUFF

Combine (1) and (2), x>x^2 and y=x^2. Since x>x^2, x^3>x^4 (multiply both sides with x^2).
Since y=x^2, y^2=x^4. => x^3>y^2. SUFF

(C)

164. Two people are working on a equation x^2+bx+c=0, the first people get two results x1=...
and x2=..., but the numbers are incorrect, they are the result for x^2+bx+d=0; the other people
get two results x1=... and x2=...,but the numbers are incorrect, they are the result for
x^2+ex+c=0. What is the correct solution of equation x^2+bx+c=0?

Although the question is not very clear, these types of questions are usually based on this concept:

For a quadratic equation ax^2+by^2+c=0,

Sum of the roots = -b/a

Product of the roots = c/a

And, any quadratic equation can be written as x^2 - (sum of roots)x + product of the roots=0.

If the question were clear, it would have been easily solvable with this concept.

165. Given 452*37*16, add 1 to which one of the 7 digits will increase the value of the product by
less than 1000?

ANS: 2

The most important thing to realize in this question is that “452” is what contributes the most to the
multiplication. When something is added to one of the numbers in the given product, suppose a is
added to 37

So new product is 452*(37+a)*16 => increase in value is 452*16*a. If you notice, the major
contributor to this value is the multiplier 452*16

So basically to minimize the increase in value, we should make the changes to 452.

So the units digit of 452 should be changed which is 2

Trick: To find the least difference in product, change the units digit of the greatest number.
166. Is x greater than 13/11?
1). X>1.182
2). X<1.19

ANS:

13/11 is easily calculated, 13/11 = 1.181818……..

Stmt 1) x>1.182 => x is definitely greater than 1.1818…. SUFF

Stmt 2) x<1.19 => x could be less than 1.1818… or less. INSUFF

167. y is not zero, is (2^x/y)^x<1?


1). x>y
2). y< 0

ANS: (E)

Given y is not zero.

Stmt 1) x>y -> best solved by values. Take y=-1 and x=1 [LHS is negative inequality is satisfied]
and take y=-1 and x=2 [LHS is positive and inequality is not satisfied]. Inconsistent result. INSUFF

Stmt 2) y<0. Take same examples as above, there is nothing given about x, so x it could be any
values. INSUFF

Combining (1) and (2), the examples still give inconsistent values. INSUFF

(E)

168. X+Y = 1; X^2 + Y^2 = 3, XY=?

ANS: XY=-1

(x+y)^2 = 1 => x^2 + y^2 + 2xy = 1 => 3 + 2xy = 1 => 2xy = -2 => xy = -1

169. A certain number of products A were sold out and the total cost was 5 percent of the sales
value plus $100,000. If the sales made a profit, is the number the products greater than 21,000?
1). The total sales value is $110,000
2). The price of the product is $5
ANS: (B)
Given: CP = .05*SP + 100000 and SP>CP

Stmt 1) SP = 110000. We can find total CP, but not the number of products. INSUFF

Stmt 2) The price of the product is $5 -> Let there be x products. So SP = 5X.

We know that SP>CP => 5X > 0.05*5*X + 100000 => X > 100000/4.75 => X>21000. SUFF

(B)

170. In the number line, is point P between Q and Z?


1). The product of P and Q is greater than 0
2). The product of P and Z is less than 0
ANS: (E)

Stmt 1) pq>0 -> either both p and q are negative or both are positive. Still we can not be sure
whether p lies between q and z because nothing is given about z. INSUFF

Stmt 2) INSUFF. Similar reasoning as above.

Combine (1) and (2) pq>0 and pz<0 => either p and q>0 and z<0 OR p and q <0 and z>0. Still
nothing can be said about their arrangement. INSUFF

(E)

171. In triangle ABC, P is a point on BC. Is AP=AB?


1) AB=BP
2). Angel APB is 60 degree

ANS: (C)

Stmt 1). Nothing can be said about AP. INSUFF

Stmt 2) Angle APB is 60 -> Nothing can be said about other angles. INSUFF

Combine (1) and (2) AB=BP and Angle APB is 60 => Angle BAP also is 60 and so is anle ABP =>
triangle ABP is an equilateral triangle. Hence AP=AB. SUFF

(C)

172. Symbol * is denoted to be X*=1/X, then (3*+2*)*=?

ANS: 6/5

3* = 1/3

2* = 1/2

=> 3* + 2* = 5/6

=> (3* + 2*)* = 6/5

173. The median of 11, 18, 43, 60 and x is 18. What is the greatest possible average of the five
numbers?

ANS: 30

Given the median is 18. The possible choices of x we have are x<11 OR x=11 OR 11<x<18 or
x=18. To find the greatest possible average we will need to use the greatest possible value of x
which is x=18.

Hence x=18 and the greatest possible average = 30

174. If the 5 times of the age of A is 10 less than 3 times of age of B, what is age of A?
1). The sum of two ages is 14.
2). A is ten year younger than B.
ANS: (D)

Given 5A = 3B-10

Stmt 1) The sum of two ages is 14 -> A+B=14. Two equations two variables. SUFF

Stmt 2) A is ten years younger than B -> A=B-10. Two equations two variables. SUFF

(D)

Discussion Link

175. X=?
1). 4^(xy) = 2^(6x)
2). 9^(y) = 3^(xy)

ANS: (E)

Stmt 1) 4^(xy) = 2^(6x) -> 2^(2xy) = 2^(6x). The possible solutions to this equation are:

X=0 (Y doesn’t matter) OR

X#0 and y=3.

So no unique value of x is known. INSUFF

Stmt 2) 9^(y) = 3^(xy) -> 3^(2y) = 3^(xy). The possible solutions to this equation are:

Y=0 (X doesn’t matter) OR

Y#0 and x=2.

No unique value of x is known. INSUFF

Combine (1) and (2), we still have two solutions wither [x=0 and y=0] OR [x=2 and y=3]. Again
unique value of x cannot be known. INSUFF

(E)

186. Each of 25 fields can produce 10 tomatoes, if farmers produce x additional fields,
each field's production will reduce by x tomatoes. If farmers produce more than 25 fields
and the total production is 174 tomatoes. How many fields did the farmers produce?

ANS: 29

Lets assume that the farmers produce x additional fields. So each fields production also
reduced by x. So total produce = (25+x)(10-x)

Given total production = 174

=> (25+x)(10-x) = 174


=> x=4.

So total fields = 29.

188. A tank is filled with gas or something like this, were there more than 30 gallons of
gas in the tank before the tank went to leak? (Note 1 gallon=128 ounces)
1). the leaking speed is 6.4 ounce per minute.
2). After it began to leak, the tank went empty less than 12 hours.

ANS: (E)

Stmt 1) Not sure how long the tank was leaking so cant say. INSUFF

Stmt 2) Since we dont know the rate we cant find the initial quantity. INSUFF

Combined (1) and (2) we know the rate and it tool less than 12 hrs to empty.

Initial capacity < 12 * 6.4 * 60 / 128 gallons

Initial capacity < 36hrs. It could be any number below 36. Cant say!

(E)

189. x is in the rectangle OABC, D is one point on side BC, what is probability of x is in
the triangle OAD? (Key: 1/2)

ANS:1/2

Theorem: Triangles with same base, with vertex a line parallel to the base have equal
area.

=> Irrespective of the position of D, the area of the triangle OAD will always be the same
= 1/2 of the are of rectangle OABC.

Probablity = (Area of Triangle OAD) / (Area of Rectangle OABC) = 1/2

190. What is the x-axis intercept of this line L?


1). L passes through point A (0,5)
2). L passes through point A (5,4)

ANS: (C)

Stmt 1) Just one point on the line can give no information about the x-intercept unless it
is a point on the x-axis itself. INSUFF

Stmt 2) Similar argument as above. INSUFF


Combined (1) and (2) Two point on a line will give the equation of the line and x-
intercept can be easily found[eqn would be x+5y=25]. SUFF.

(C)

191. In a class, 70% of the student choose geometry, 80% of the student choose physics,
what is the least probability of students choose both geometry and physics?

ANS:

There could be some students who choose neither geometry nor physics = x

T - x = P + G - PnG

PnG = P + G - (T-x)

(PnG)min = P + G - (T-x)max

(T-x)max will occur only when x=0.

(PnG)min = P + G - T = 50

Probablity = 0.5

192. Is a-3b an even number?


1). b=3a+3
2). b-a is an odd number

ANS: (D)

Stmt 1) b=3a+3 -> a-3b = a-(3a+3) = a-3a-3 = -2a-3 = -(2a+3) -> [even+odd] Definitely
odd. SUFF

Stmt 2) b-a is odd. a-3b = -(3b-a) = -(2b + b-a) = - (2b+{b-a}) -> [even + {odd}].
Definitely odd. SUFF

(D)

193. 4000 gallon can fill the capacity of a tank from 3/4 to 4/5, what is the full capacity
of the tanker?

3/4L - 4/5L = 4000

-0.75L + 0.8L = 4000

0.05L = 4000
L = 4000/0.05

L = 80,000

194. A sequence of numbers is 1/2, 1/4, 1/8, 1/16, 1/32, what is the range of the tenth
number a10 in the sequence?
Answer: 0.0001<a10<0.001

ANS: 0.0001 < a10 < 0.001

Tenth term = 1/2^10 = 1/1024

1/10000 < 1/1024 < 1/1000

=> 0.0001 < a10 < 0.001

Tutors answers

1. A 12 feet x 8 feet wall is completely covered with some square tiles. A figure shows that each tile
has 16 small squares, including 4 blue and 12 white. What is the total number of the blue squares
on the wall?

1 feet = 12 inch, I think the answer should be: (12*8*12*12/4*4)*4

Answer:

The area of the wall is 12*8, in foot. Switched to inches, the area became 12*8*12*12.

Given that the area of the tile is 4*4, in inches. Then, the number of tiles on the wall is
12*8*12*12/4*4.

Each tile has 4 small blue squares, then, the number of the blues is:

(12*8*12*12/4*4)*4

2. A mechanic system consists of a large gear and a little gear. The rating life of the gear is 60,000
rounds. What is the difference between the large gears’ life and the little gears’ life? [The two gears
joggled to each other?] [The two gears joggled to each other?]

1). The ratio between two radius is …

2). It cost 2 minutes for the little gear to rotate one circle.

Answer:

From statement 2, we can know that the life of the little gear = 60,000*2, in minute.

Then, what is the life of the large gear?

Let R, r be the radius of two gears, V, v be the speed (circles/minute) of two gears.
Noticed that the two gears joggled to each other (assume that it is a true condition),we get,
2pi*R*V=2pi*r*v, hence, V/v=r/R, where r/R is given by statement 1, and v is given by statement
2, then V come out.

The life of large gear =60,000/V

Hence with the both the statements together, we can answer the given question.

3. 50 students of a certain class took a test. How many of them got a score less than 82?

1). the average score is 82

2). the median is 82

Answer:

Statement 1 is insufficient.

Statement 2, the distribution of scores could be: 24 scores less than 82, then 82, 82, then 24
scores greater than 82. Or, 25 scores less than 82, including the largest score 81, and 25 scores
greater than 82, including the least score 83. Insufficient.

Answer is E

4. Sequence s1, s2, s3,..sn, is such that s=1/n-1/n+1 . Is the sum of first n terms greater than
9/10?

1). n>10

2). n<19

Answer:

Given that, s1=1/1-1/2, s2=1/2-1/3, s3=1/3-1/4, …sn=1/n-1/(n+1)

Let Sn be the sum of the first terms, then,

Sn=1-1/2+1/2-1/3+1/3-1/4+…1/n-1/(n+1)=1-1/(n+1).

For Sn>9/10,

1-1/(n+1) must be greater than 9/10, or n>9.

According that, statement 1 alone is sufficient, statement 2 alone is insufficient.

Answer is A

5. Is x-y>0? (Or x+y>0?)

1). x-y/x>0

2). x+y/x-y>0

Answer:
Before new information come in, we think it’s E

6. [(4^300+2^600)/2]^4=?

Answer:

[(4^300+2^600)/2]^4=[(2^600+2^600)/2]^4=(2^600)^4=2^2400

7. The average of some numbers is 43, what is the standard deviation?

1). The least number is 43

2). The greatest number is 43

Answer:

1). Given that the least one is equal the average, then all the numbers are the same.

2). Given that the greatest one is equal the average, then all the numbers are the same.

Statement 1 and 2 each alone is sufficient.

8. S(n)=S(n-1)^S(N-2), S1=1,S2=2,S3=2^1....S6/S5=?

Answer:

S1=1

S2=2

S3=2^1=2

S4=2^2=4

S5=4^2=2^4

S6=(2^4)^4=2^16

S6/S5=2^12

9. The sides of a triangle is a, b, and c. Are the three angles all less than 90 measure degrees?

1). The areas of the semi-circles with the radius a, b, c are 4, 5, 6, respectively.

2). c<a+b<c+2

Answer:

Dan82 gave an excellent A&E in the following page. For you reference:

http://www.scoretop.com/forum/forum_posts.asp?TID=28961& PN=1
10. Is x divisible by 3?

1). When x is divided by 5, the remainder is 1

2). When x is divided by 15, the remainder is 1

Answer:

1). Given that x could be 6, 11, 16, 21,… where 6, 21 are divisible by 3, but 11 and 16 are not
divisible by 3. Statement 1 alone is insufficient.

2). Given that x could be 1, 16, 31, 46, …none of them could be divisible by 3. Statement 2 alone is
sufficient.

Hence, answer is B.

11. There are two points p(a,b), and q(c,d) in a xy-plane. If a^2+b^2+c^2+d^2=87, and
ac+bd=31, what the length of the segment pq?

Answer:

PQ=[(a-c)^2+(b-d)^2]^1/2=(a^2+b^2+c^2+d^2-2ac-2bd)^1/2=(87-62)^1/2=5

12. In a certain survey, each of 800 people expressed their attitude to A and B. They can say yes,
no, or unsure. If 200 people said yes to A but not to B, how many people said yes to either A or B?

Yes No Unsure

A 500 200 100

B 400 300 100

Answer:

Given that 500 say yes to A, and 200 say yes to A but not to B, then 300 say yes to both A and B,
then 400-100 say yes to only B.

So, 200(Only A)--300(A&B)--100(Only B)

Then, the number of people said yes to either A or B is 200+300+100=600

[If the question is: how many people say yes to only A or only B? the answer is 200+100=300.
Please check it carefully when meet it]

13. This semester, each of the 90 students in a certain class took at least one course from A, B, and
C. If 60 students took A, 40 students took B, 20 students took C, and 5 students took all the three,
how many students took exactly two courses?

Answer:
With a figure we can simplify such questions.

As the figure shows, what we want to know is the area of the blue region.

The area of the whole region is 90 (given that every one will enroll at least one course).

We also can get the area by: 60+40+20=120.

Why the area is greater than 90, the real area? Cause we count the red region for three times and
the blue region for two times.

Therefore, to get the real area, 120 must subtract blue once and red twice. That is:

120-blue-2*red=90, red region is 5.

So, the blue area =120-90-10=20

14. X, y and z are three digits numbers, and x=y+z. Is the hundreds’ digit of x equal to the sum of
hundreds’ digits of y and z?

1). the tens’ digit of x equal to the sum of tens’ digits of y and z

2). the units’ digit of x equal to the sum of units’ digits of y and z

Answer:

To answer it, the most important information we need to know is whether there is carry from tens’
digit.

According to the statement 1, there is no carry from tens’ digit. Knowing that x is three digits
number, then, its hundreds’ digit is sum of that of y and z.

Statement 2 gives no information about the carry from tens’ to hundreds’ digit.

Answer is A

15. If a=x+1/x, b=x-1/x, 2^(a^2)/2(b^2)=?

Answer:

Given that a^2=x^2+2+1/x^2


b^2= x^2-2+1/x^2

So, 2^(a^2)/2(b^2)=2^(a^2-b^2)=2^4=16

Or:

2^(a^2)/2^(b^2)=2^(a^2-b^2)=2^[(a+b)(a-b)]=2^4=16

16. In the first half of a certain trip, a car maintained an average speed of 40m/h; in the second
half of the trip, the car maintained an average speed of 60m/h. What is the average speed for the
whole trip?

Answer:

Let 1 be the whole trip.

Then, the first half of the trip cost time=0.5/40, the second half of the trip cost time=0.5/60.

So, the average speed for the whole trip is 1/(0.5/40+0.5/60)=48m/h

17. Is |x-2|+|x+2|<4?

1). x<0

2). x is within (-2, 2)

Answer: The points of interest are x=2, x=-2.

(A) x=0 => x<2 but nothing can be said about x and -2.

case 1: x>=-2 => |x-2|+|x+2| = 2-x + x+2 = 4 ..... so given inequality reads is 4<4? -> NO

case 2: x<-2 => |x-2|+|x+2| = 2-x -2-x = -2x since x<-2, -2x>4 => |x-2|+|x+2| > 4 => is |x-2|+|
x+2|<4 ? -> NO

Both give NO as answer. Consistent. SUFF

(B) x lies between (-2,2) => -2<x<2 => |x-2|+|x+2| = 2-x+x-2 =4. NO. SUFF

Hence (D)

18. The area of a triangle is 16*root3. What is the perimeter of the triangle?

1). It is an isosceles triangle

2). Two of the inside angles are 30 degrees.

ANS: (B)

Stmt 1) Given area = 16sqrt(3) and triangle is isosceles. Let b=base and h=height. From given
1/2*b*h=16*sqrt(3). We have 2 variables 1 equation. INSUFF
Stmt 2) Given area = 16sqrt(3) and two inside angles are 30. This implies this triangle is also
isosceles, and there is extra information available (angles = 30,30,120). Let b be the base, h be the
height and s be the measure of two equal sides. We have following relations

Perimeter = b+2*s, Area = 1/2*b*h, h*tan(30)=b/2, s*cos(30)=b/2 (4 equations and 4


variables). Solvable. SUFF

(B)

19. As the figure shows, A is the center of the circle, AB=AC=2, angle BAC is 120 degrees. What is
the area of the triangle?

ANS: sqrt(3)

Draw a perpendicular AD to BC, AD bisects BC. Angle B = Angle C = 30.

Height = ABsin 30=1, base = ABcos30+AC cos30 = 2ABcos30=2¡î3.

Area = 1/2*base*height = 1/2*2*sqrt(3) = sqrt(3)

20. The median of the rent of the apartments is 550. What's the average (arithmatic mean) of the
rent?

1). The average rent of apartments with rent higher than the median is 890.

2). There are 370 apartments with the rent higher than the median.

ANS: E
Median is basically the middle value when the rents (or items) are arranged in increasing (or
decreasing) order. Average rent is (total sum of rents)/(total number of apartments). The
question just gives us the median value.

Stmt 1) Average rent of apartments with rent higher than the median is 890 -> Gives gives us an
average of upper half. We still have no information about the total number and total sum of
apartments. INSUFF

Stmt 2) There are 370 apartments with the rent higher than the median -> Gives just the number
of apartments greater than the median. No information about rents. INSUFF

Combining 1 and 2. We can find the total sum of the upper half = 890*370. Still no clue about the
sum of rents of houses below the median (lower half) => INSUFF

(E)

21. What is the area of the circle inscribed in the square with side 7?

ANS: (3.5)2*pi

2*radius = side of square => radius = 7/2 = 3.5.

Area = pi * r2 => (3.5)2*pi

22. Two cubes each is numbered with 1 to 6 (called dice). Tossing the dices, what is the probability
that the sum of the numbers on the top is greater than 9?

ANS: 1/6

Total possibilities = 36

Favorable outcomes = (4,6), (6,4), (5,6), (6,5), (5,5), (6,6) = 6

Probability = 6/36 = 1/6

23. The first term of a geometric sequence is 1/2, and the common ratio is 1/2. The 10th term will
fall into which of the following range?

ANS: (C) 0.001 ~ 0.0001

0.1~0.01, 0.01~0.001, 0.001~0.0001,0.0001~0.00001

1st term is 1/2 so 10th term would be (1/2)10 = 1/1024.

(1/1000)>(1/1024)>(1/10000) [just increasing the denominators, fraction decreases] =>


0.001>(1/2)10>0.0001. (C)
24. The sides of a triangle is a, b, and c. Is it a right triangle?

1). a^2+b^2=3^2+4^2

2). a^2+c^2=3^2+5^2

ANS: (E)

*Note : a,b and c are NOT given to be integers

Stmt 1) a^2+b^2=3^2+4^2=25 -> Nothing is given about c and its relation with a and b. INSUFF

Stmt 2) a^2+c^2=3^2+5^2=34 -> Nothing is given about b and its relation with a and c. INSUFF

Combining (1) and (2), we have 3 variables (a,b,c) and just 2 equations. There are multiple values
possible for a, b and c which satisfy these equations. INSUFF

(E)

25. -1<x<-3/4, which of the following has the greatest value?

-x, -1/x, 1-1/x, 1/(1+x)

ANS: 1/(1+x)

Minimum value of x = -1

Maximum value of x= -3/4

Substituting these values in the given expressions provides the extremities (range) of the
expressions.

1/(1+x) goes to infinity when x=-1, while others all have finite maximum values. Hence 1/(1+x) will
have the greatest value.

26. A list of consecutive natural numbers contains 9 and 15. Does the list contain number which can
be divisible by 8?

1). The list has 10 numbers.

2). 18 is in the list.

ANS: (D)

Stmt 1) The list has 10 numbers -> the list will contain either 8 or 16 (at the minimum). SUFF

Stmt 2) 18 is in the list -> the list contains 16. SUFF

(D)
27. In the xy-plane, line A has slope a and line B has slope b. Are the two lines parallel to each
other?

1). 4^a=2^b

2). 2^a=3^b

ANS: (C)

For the 2 lines to be parallel a=b. So basically the question is asking if a=b?

Stmt 1) 4^a = 2^b => 2^2a = 2^b => 2a=b. Cant be sure, because if a=b=0, lines are parallel
and for non-zero values of a and b they are not parallel. INSUFF

Stmt 2) if 2^a=3^b then 2^(a/b)=3. Now we know that 2^1=2 and 2^2=4. so the value of (a/b)
should lie between 1 and 2 so that 2^(a/b)=3.

Hence there is atleast one more (non-integer) value of a and b (apart from a=b=0) which would
satisfy the given equation. Hence statement 2 is not sufficient in itself (gives multiple values of a
and b). INSUFF.

Combining (1) and (2), the only possible value for a and b is 0 (Also there are 2 equations and 2
variables). Substitute 2^a=3^b in stmt 1, we get 9^b = 2^b. This is only possible for b=0 (=>
a=0). Hence a=b=0 => Parallel. SUFF

(C)

28. If x and y are nonzero, is (x^3)(y^5)/(x^2)(y^3)>0

1). x>0

2). y>0

ANS: (A)

The question is basically asking is xy^2>0?

Stmt 1) x>0 => xy^2 is definitely greater than 0 sine y^2 is always greater than 0. SUFF

Stmt 2) y>0 => Doesn’t tell us anything about sign of x. INSUFF

(A)

29. Each segment from A to G has equal distance. Which of the following point represents [-
(3^12)] ?

ANS: Point D

Distance between all points is same. Given F as 3^11 and G as 3^12. So distance between 2 points
= 3^12 – 3^11 = 3^11*(3-1) = 2*3^11.

So point E = F – 2*3^11 = 3^11 – 2*3^11 = -3^11. Point D = Point E – 2*3^11 = -3^11 –


2*3^11 = -3*3^11 = -1*3^12 = -3^12
30. A parallelogram with area of 18 has a side of 6. Which of the following could be one of its inside
angles?

30, 50, 60, 70, ...

SORRY! Data insufficient. There is no point assuming what the missing data could be! . The
concept though is fairly straightforward (area of parallelogram etc).

31. A ship started off at the rate of a. t hours later, in the same direction, another ship started off at
the rate of b. In how many hours the second ship will catch up with the first ship?

ANS: a*t/(b-a)

Method 1: Speed Time Distance

Lets assume that after t1 hrs, 2nd ship catches the first.

Since the 2nd ship catches up, the total distance traveled by second is equal to total distance
traveled by the first.

Hence b*t1 = a*t + a*t1 => t1 = a*t/(b-a)

Method 2: Relative speed

Initial distance between them = a*t.

Relative speed of 2nd ship w.r.t 1st ship = b-a.

Time taken to cover the distance = initial distance / relative speed = a*t/(b-a)

32. In the triangle ABC, angle A is 30 degrees, and BD is perpendicular to AC. What is the
perimeter of the triangle? (changed the question, since AD per to AC looks typo)

1). given the perimeter of the triangle ABD.

2). Given the perimeter of the triangle BCD.

ANS: (C)

Question asks perimeter of ABC = AB+BC+CA.

Stmt 1) perimeter ABD = p (known value). => AB + BD + DA = p. Since ABD is a 30 60 90


triangle, AB:BD:DA = 2: 1 : sqrt(3). From this we can find the values of AB, BD, DA. But we know
nothing about CD so we cannot find CA. INSUFF

Stmt 2) perimeter BCD = p1 (known value). Not much information except angle BDC is given.
INSUFF

Combining (1) and (2), AB, BD, DA are known. Also BC+CD+DB = p1 => BC+CD = p1-DB (comes
to known value). Perimeter of ABC = AB+BC+CD+DA. We already know the values of AB and DA,
and the sum of BC+CD is also known. Hence Perimeter can be found. SUFF

(C)
33. What is the units' digit of (3^11)(4^13)+1?

ANS: 9

The units digit of “powers of 3” follow the repeated pattern 3, 9, 7, 1. Units digit of 3^11 (same as
units digit of 3^3) = 7.

Units digit of “powers of 4” always follow the pattern 4,6,4,6 (even powers 6 and odd powers 4).
Hence units digit of 4^13 = 4.

Units digit of 3^11 * 4^13 + 1 = units digit of 4*7 + 1 = 8 + 1 = 9

34. Some toys include large, middle, and small model with red, yellow, green, or blue color. If
numbers of all model-color combinations are the same, for example, number of red large toys is
equal to number of green little toys. A boy wants a red-large toy. If his mother select one for him at
random, what is the probability that at least one of the color and model will satisfy the boy?

ANS : 1/2

For each color, let there be x large, x medium and x small toys. So there are total “3x” each of red,
yellow, green and blue colored toys.

Total toys = 4*3x = 12x.

The boy wants red-large toy. The question asks probability that “at least one of the color and
model will satisfy the boy”. So either a red or a large toy.

So total number of favorable toys (satisfying atleast 1 condition of the boy)= 3x (red toys) + 3*x (x
large-blue, x large-yellow, x large-green) = 6x toys.

Probability = 6x/12x = 1/2

35. Is point C has the same distance from point A(-3,-3) and point B(1,-3)?

1). C lies on the line x=-1

2). C lies on the line y=-3

ANS: (A)
Stmt 1) C lies on the line x=-1. All points on this line (as given in the diagram) have the same
distance from A and B. Each point C on line x=-1, forms an equilateral triangle with points AB. SUFF

Stmt 2) C lies on the line y=-3. Point (-1,-3) lies in the middle of And B all other points are at
unequal distance. So cant say. INSUFF

(A)

36. One of the solutions of x^2-5x+t=0 is -1. What is the other solution?

ANS: x=6

Substitute x=-1 in the equation. 1+5+t=0 => t=-6. So the original equation is x^2-5x-6=(x+1)(x-
6). So the other solution clearly is x=6.

37. If 3^a*4^b =576, where a and b are integers, 4^a*3^b=?

ANS: 432

Prime factorize 576 (since a and b are integers, this is the best approach). 576 = 3^2 * 2^6 =
3^2*4^3 => a=2, b=3.
4^a*3^b = 4^2*3^3 = 432.

38. The area of a triangle with the sides x and y is yx/2. What is the angle formed by x and y?

ANS: right angle

Area = 1/2 *base * height. Since area is equal to yx/2, y and x will be base and height (or vice-
versa). So two sides are perpendicular to each other =>they have right angle between them.

39. People A and B together can finish 800 products in x hours. If it takes y hours for A alone to
finish the 800 products, how many hours will it take for B alone to finish the 800 products?

ANS : xy/(y-x)

A takes y hrs to finish the work. B takes z hrs to finish the work. We have to find z.

In 1 hr A will be done with 1/y amount of work, B will be done with 1/z amount of work and as
given, if they work together, they finish 1/x of work (in 1 hr) => 1/y+1/z=1/x

1/y+1/z=1/x

1/z=1/x-1/y => z = xy/(y-x)

40. Tickets for a movie include A, with rate of $4.5, and B, with rate of $6. If 60% of the tickets
sold are tickets B, what percent of the total revenue on tickets sales are from B?

ANS: 66.7%

Let T be the total number of tickets sold.

So revenue from sales of B = 6*0.6T and revenue from A = 4.5*0.4T.

Percentage = 6*0.6T/(4.5*0.4+6*0.6)T = 3.6T/5.4T * 100 = 3.6/5.4 * 100 = 66.7%

41. After every 45 minutes trip, a car will take a 15 minutes break. In highway, the car¡¯s rate is
60m/h, and in country road, the rate is 40m/h. In a 4 hours trip, the car can travel how many more
miles in highway than in country road?

ANS: 60 miles

The question basically asks that if car were to travel 4 hrs on either highway or country road, what
will be the difference in distance traveled. (I think there are other possible interpretations of the
questions as well, but the concept is fairly simple here, Speed Time Distance. If in your
interpretation you find data as sufficient and solvable, then it should be fine.)

Car basically travels for 45 minutes in 1 hr. So in 4 hrs trip it will travel for 45*4 minutes = 180min
= 3 hrs.

Distance traveled if it goes on highway = 60*3 = 180 miles

Distance traveled if it goes on country road = 40*3 = 120 miles

Difference = 180-120 = 60 miles


42. How many ways are possible to arrange A, O, L, L, and P with two "L" being separated by at
least one letter?

ANS: 36

Method 1 (preferred):

Total number of ways 5 letters (with 1 letter repeated twice) can be arranged among themselves=
5!/2!

Total number of ways 5 letters can be arranged among themselves, if both L’s stay together
(effectively arranging 4 letters) = 4!

Total number of ways of arranging the letters such that L’s are not adjacent = 5!/2! – 4! = 36

Method 2:

Consider 5 places to be filled by these letters such that two L’s are not adjacent.

(a) Put 1st L in the first place (the other L cannot be on the second place. It can be on the 3rd, 4th or
5th place)

L _ L _ _ (two positions are fixed, remaining 3 can be arranged in 3! Ways)

L _ _ L _ (Again, two positions are fixed, remaining 3 can be arranged in 3! Ways)

L _ _ _ L (Again 3! Ways)

Total ways = 3*3!

(b) Put 1st L in second place (The second L can only be in 4th and 5th place) => Applying above
logic, total ways = 2*3!

(c) Put 1st L in third place (the second L can only be in 5th place) => total ways = 3!

Total = (a)+(b)+(c) = 6*3! = 6*6=36

43. If an=3a(n-1)-x, and a5=99, a3=27, x=?

ANS: x=36

a5=3a4-x

a4=3a3-x => a5=3*(3a3-x)-x => a5=9a3-3x-x => a5=9a3-4x. Substituting, x=36.

44. 32^5=2^x, x=?

ANS: x=25

32^5 = (2^5)^5=2^(5*5)=2^25 => x=25.


45. If A and C represent the area and the circumference of a circle, respectively, and A:C=1:2root2,
A=?

ANS: pi/2

A=pi*r^2, C=2*pi*r

=> A:C = r:2, given A:C = 1:2sqrt(2) => r = 1/sqrt(2)

Area = pi*r^2 = pi/2.

46. Of a group of people, the ratio of number of women to men to children is 7 to 2 to 5. What is
the total number of the people?

1). women + children=12

2). number of men is less than 4

ANS: (D)

Given W:M:C = 7:2:5 => W=7x, M=2x, C=5x

Stmt 1) women + children=12 -> W+C=12 => 12x=12 => x=1 (Only possible value). Hence W=7,
C=5 and M=2(Only integer values which satisfy the given ratio and condition). SUFF

Stmt 2) number of men is less than 4 -> Given W:M:C = 7:2:5. Given M<4 => 2x<4 => x<2 =>
x=1(only possible value). Hence the total number of people=14. SUFF

(D)

47. Of the people in a banquet, if 75% chose dissert, what percent chose coffee?

1). 60% of the people who chose dissert also chose coffee.

2). 90% of the people who chose coffee also chose dissert.

ANS: C

Let x be the number of people who chose dessert. Given x=0.7T

Let y be the number of people who chose coffee

Let z be the number of people who chose neither dessert nor coffee

T = x+y+z

Stmt 1) 60% of the people who chose dissert also chose coffee -> 0.6x chose coffee. But nothing is
known about y or z. Hence INSUFF

Stmt 2) 90% of the people who chose coffee also chose dissert -> 0.9y chose dessert. But nothing
is given about x or z. INSUFF

Combining (1) and (2) 0.6x=0.9y => 0.6*0.75T = 0.9y => y=0.5T => y=50%. SUFF
(C)

48. When integer n is divided by 3, the remainder is 2. Is the integer divisible by 5?

1). When divided by 45, the remainder is 30--the conditions seems incorrect.

2). The integer is divisible by 2

ANS: (A)

I don’t see anything wrong with the condition 1 as such, hence taking it as it is.

Given n=3k+2.

Stmt 1) When divided by 45, the remainder is 30 -> n=45x + 30. n is clearly divisible by 5. SUFF

Stmt 2) Integer is divisible by 2 -> n=2x. Since n is even and n=3k+2, 3k must be even too. Two
numbers which clearly satisfy these conditions are: 26,50. One is divisible by 5 other not. INSUFF

(A)

49. In the xy-plane, both line K and L intersect with axis-y. Is K’s intercept with axis-y greater than
that of line L?

1). K’s intercept with axis-x is greater than that of L.

2). K and L have the same slope.

ANS: (E)

Note: For those in confusion: When a question talks about y-intercept, the sign matters. Y-intercept
can be positive or negative.

Stmt 1) intercept with axis-x is greater than that of L -> Nothing can be said about the y intercept.
INSUFF

Stmt 2) K and L have the same slope. Let K be y=mx+c1 and L be y=mx+c2, where c1 and c2 are
y-intercepts. Nothing can be said about c1 and c2. INSUFF

Combining (1) and (2) Given “X intercept of K” > “X intercept of L” (put y=0 in above equations)=>
-c1/m > -c2/m => c1/m < c2/m. We cannot take “m” off this inequality unless the sign is given
(depending on the sign the inequality may reverse). Hence nothing can be said about c1 and c2.
INSUFF

(E)

50. The range of set A and B combined is 600. If range of A is 200, what is the range of B?

1). ...

2). The least number in B is 100 greater than the largest number in A
ANS: (B)

Stmt 1) What can I say! 

Stmt 2) Given range of A and B combined is 600. Also given that least number in B is greater than
largest number in A, therefore

Range of A and B combined = B(Largest) – A(smallest) = 600 … (1)

A(largest) – A(smallest) = 200 (given in question) …(2)

B(smallest) - A(largest) = 100 (condition of stmt 2) …(3)

(1) – (2) – (3) = B(largest)-B(smallest) = 600-200-100 = 300 (Range of B). SUFF

(B)

51. Is y>0? The question is somewhat similar to Q5.

1). (x-y)/x > 0

2). (x^2-y^2)/x^2>0

ANS : (E)

Stmt 1) (x-y)/x > 0 ->Since nothing is given about x, it could be positive or negative. There are two
cases:

x<0 => (x-y) <0

x>0 => (x-y) >0

INSUFF

Stmt 2) (x^2-y^2)/x^2>0 -> Since x^2 is always positive => x^2-y^2>0 => (x-y)(x+y)>0. Since
nothing is know about x or y we cant say anything about sign of y. INSUFF

Combined (1) and (2) we have two cases:

x<0 => (x-y)<0 and hence x+y<0 (since (x-y)(x+y)>0). Since x<0 and x<y and x+y<0, this is
possible for positive and negative values of y. Cant say anything about y. We don’t need to go
further. INSUFF

(E)

52. If x<y<z and x, y, z are integers, which of the following must be greater than y^x+y^z?

1). x ^ (y+z)

2). y^2z

ANS: None of the given.


Stmt 1) x ^ (y+z) -> x^y*x^z. No direct relation can be determined between the two equations.
The best way is to plug in values (x=-2, y=0, z=1). INSUFF

Stmt 2) y^2z -> Again the best method is to plug in the values. Plugging -2, 0, 1, and (1,2,3) we
get in consistent answers. INSUFF

53. Three machines can finish a certain work in 36 hours at the same rate. If a 4th machine with
the same rate is added in, what is the time to finish the work?

ANS: 27 hrs

Three machines can finish a certain work in 36 hrs => 1 machine itself finishes the work in 36*3
hrs.

Four machines will take 36*3/4 hrs of time = 27 hrs.

54. F(x)=1+1/x, F(F(x))=?

ANS: (2x+1)/(x+1)

(F(F(x)) = 1 + 1/F(x) = 1 + 1/(1 + 1/x) = 1 + x/(1+x) = (2x+1)/(x+1)

55. S is the set of all solutions of equation (x-1)(x+2)+(x-1)(x+4)=0. Which of the following must
be true?

ANS:

(x-1)(x+2)+(x-1)(x+4)=0 => (take x-1 common) (x-1) [x+2+x+4]=0 => (x-1)[2x+6]=0 => (x-
1)(x+3)=0

=> x=1, x=-3

56. (r^2-s^2)^2=?

1). 4r^2s^2=120

2). (r^2+s^2)^2=4r^2s^2

ANS: B

Question basically asks what is the value of r^4+s^4-2r^2s^2.

Stmt 1) 4r^2s^2=120. Clearly Insuff.

Stmt 2) (r^2+s^2)^2=4r^2s^2 -> (r^4+s^4+2r^2s^2)=4r^2s^2 -> r^4+s^4-2r^2s^2=0 =>


(r^2-s^2)^2=0. SUFF

57. If a and b are none negative integer, ab=?

1). 4^a=2^b

2). 2^a=3^b

ANS: (B)
Stmt 1) 4^a=2^b -> 2^2a=2^b => 2^(2a-b)=1 => 2a-b=0 or 2a=b. Nothing can be said about
ab. INSUFF

Stmt 2) 2^a=3^b. Since a and b are integers, a=b=0 => ab=0. SUFF

(B)

58. What is the remainder when 9^845 is divided by 10?

ANS: 9

The units digit of 9^845 will be the remainder when it is divided by 10. Units digit of 9^x is 9 or 1,
depending on whether x is odd or even (respectively). Since 845 is odd, the units digit of 9^845 will
be 9. Hence remainder is 9.

59. Two buildings are 10m and 12m high, respectively. If the distance between the bottoms is 6,
what is the distance between two building tops?

ANS: sqrt(40)

As given in the figure, the distance between the tops is sqrt(AE^2 + EF^2) = sqrt(2^2+6^2) =
sqrt(40).

60. ----P---R----M----S-----

In the number line, R is zero, M is midpoint between P and S, and PR=1/3PS. What is P?

1). M is 1.5

2). S is 6

ANS: (D)

Stmt 1) M is 1.5 -> Given PS=3PR and PM=MS=PS/2 ( M is midpoint of P and S). Given RM=1.5
=> PM-PR=1.5 => PS/2-PR=1.5 => 3/2*PR-PR=1.5 => 1/2*PR
Stmt 2) S is 6 -> RS=6. PR+RS=PS. Given PS=3PR => PR+6=3PR => 2PR=6 => PR=3. Hence
P=-2. SUFF

(D)

61. The average of 20 numbers is 82. How many numbers are equal to 75?

1). Three numbers are between 75 and 82

2). The median is 85

ANS: E

Stmt 1) Three numbers are between 75 and 82 -> Doesn’t really tell us anything about how many
numbers are equal to 75. We can have any combination of numbers to get the average to 82 (with
given conditions). INSUFF

Stmt 2) The median is 85 -> All this tells us that 85 is the middle value. There may be 0 or more
numbers equal to 75.

Combining (1) and (2) we get the information that there are three numbers between 75 and 82, the
median is 85. However we can come with a lot of combinations with different numbers equal to 72.
INSUFF

(E)

62. n=?

1). The tens digit of 11^n is 4

2). The hundreds digit of 5^n is 6

ANS: (E)

Stmt 1) 11^4 = 14641, 11^14 will also have 4 in the hundreds place, similarly 11^24 11^34 etc.
No unique value for n. INSUFF

Stmt 2) 5^4=625, 4^14 will also have 4 in the hundreds place, similarly n=14, 24. So no unique
value for n. INSUFF

Combining (1) and (2), doesn’t help us much, n=4, 14, 24. INSUFF

(E)

63. With a speed of 20m/h, a truck will passes through a 4024 feet bridge in approximate how
many minutes?

ANS: 2.3 mins

1 mile = 5280 feet

20m/hr = (20 * 5280)/60 feet/min.

Time taken = 4024/ (20*5280/60) = 2.3 min


64. When integer n is divided by 9, the remainder is 3. Is the integer divisible by 5?

1). When n is divided by 45, the remainder is 30

2). n is divisible by 2

ANS: Same as 48

65. In the figure shown above, ABCD is a rectangle, CD=2, OCD is an isosceles triangle. Is OCD an
equilateral triangle?

1). Angle ADO=30 degrees

2). The area of ABCD is 2 root 3

ANS: (D)

Stmt 1) Angle ADO=30 => Angle ODC=60=Angle OCD (Given the Triangle is isosceles). Since two
angles are 60 degrees, third has to be 60 degrees. So all angles are 60 degrees => equilateral
triangle. SUFF

Stmt 2) The area of ABCD is 2 root 3 -> Area of rectangle = 2*sqrt(3)= DC*BC=2*sqrt(3)=2*BC
=> BC=sqrt(3) => height of the triangle is sqrt(3) => Angle ODC=Angle OCD = 60. This implies
Triangle is an equilateral. SUFF

66 In the figure shown above, ABC is a right triangle, and three rectangles are square. What is the
area of the largest square?

1). The sum of area of two small squares is given.

2). The area of the triangle Is given.

ANS (A)

Stmt 1) The sum of area of two small squares is given -> We have the sum of the squares of the
sides of right angled triangle => we can obtain the square of the hypotenuse of the triangle which
gives us the area of largest square. SUFF

Stmt 2) The area of the triangle Is given -> Doesn’t give any information about sides of triangle.
Cant say anything about of the area. INSUFF

(A)

67. Each of the 50 people chose one dish from A, B, and C, where rates of A, B, and C are $15,
$12, and $18, respectively. If the total cost was $480, how many people chose C?

1). The number of people who chose A was 12 greater than number of people who chose B.

2). The number of people who chose A was two times the number of people who chose B.

ANS (D)

The question gives two equations: A+B+C=50 and 15A+12B+18C=480


Stmt 1) The number of people who chose A was 12 greater than number of people who chose B =>
A=B+12 => We now have equations three variables, we should be able to solve for C. SUFF

Stmt 2) The number of people who chose A was two times the number of people who chose B =>
A=2B => We have 3 equations in 3 variables, we should be able to solve for C. SUFF

I could see that we get decimal values for A,B,C (they should be integers), this may be due to
incorrect values provided in the question. However the concept is straightforward.

(D)

68. Is X^2 < X^3?

1). X<X^2

2). X<1

ANS (B)

The given inequality is only true for x>1.

Stmt 1) x<x^2 -> This inequality holds for positive numbers and negative numbers. However the
inequality, x^2<x^3 holds only for x>1. we will get inconsistent answers to the asked question.
INSUFF

Stmt 2) x<1 -> Clearly, we can answer the question as NO. SUFF

69. The sales price of a certain item is D, and the sales tax is P percent. If someone paid $20 for
the item and got change of C, is the sales price greater than $15?
1). C<5
2). P=6

ANS (E)

The question basically asks if D>15?

Stmt 1) C<5 -> This means that the person paid more than 15$ in all for the item. However we are
not sure about D or P. INSUFF

Stmt 2) P=6 -> This means the sales tax is 6%. If D was 18, the total price would be 19.08, if D
was 14, sales price would be 14.84. Since C could be anything, we cannot have an idea about D.
INSUFF

Combining (1) and (2) C<5 and P=6. If D=14.5, total price=15.37 (=>C<5). If D>=15 also can get
permissible values of C less than 5. So basically we have inconsistent values for D. INSUFF

(E)

70. Is 3x+1 divisible by 10?


1) x = 4n+2
2) x > 4

ANS (A)
Stmt 1) x=4n+2 -> 3x+1 = 3(4n+2) + 1 = 12n+6+1 = 12n+7. For a number to be divisible by 10,
the units digit should be a 0. In case of “7” it is possible only when 7 is added to a number with
units digit as 3. Since 12n is an even number, the units digit for 12n can never be 3. Hence 3x+1 is
never divisible by 10. SUFF

Stmt 2) x>4, x could be 13 or 14 => 3x+1 may or may not be divisible by 10. INSUFF

(A)

71. Is 2^[(x/y)x]<1?
1). x < 0
2). y < 0

ANS (C)

The question basically asks if 2^(x^2/y)<1. Which can happen when x^2/y<0. Since x^2 is always
positive (other than for x=0), The inequality will hold when x#0 and y<0.

Stmt 1) x<0. Clearly INSUFF

Stmt 2) y<0. Almost SUFF, but there is a possibility of x=0. INSUFF.

Combined (1) and (2) we have both conditions satisfied (x#0 and y<0). SUFF

72. One unit of fat contains 9 calorie, one units of B contains 4 calorie, one unit of C contains 4
calorie. If a snack consists of 15 fat, 16 B, and 4 C, what percent of the calorie contained by the
snack fat is provided by the fat?

ANS: 62.7%

Percentage = Calorie from fat/total calories *100 = 15*9/(15*9+16*4+4*4) *100 = 135/215 *100
= 62.7%

73. A people walked from signpost 1 to signpost 2 at the average rate of 0.125km/m. Is the
distance between two signposts greater than 0.8km?
1). It cost more than 400 second for the people finish the trip
2). It cost less than 450 second for the people finish the trip.

ANS: (A)

Stmt 1) It cost more than 400 second for the people finish the trip -> Distance traveled in 400 secs
= 0.125/60 * 400 = 0.83 km. Since the trip took more than 400 second, we can be certain that the
people walked more than 0.8km. SUFF

Stmt 2) It cost less than 450 second for the people to finish the trip. The time could be anything
from 0 to 400 sec. Hence, people could have traveled any distance <= or > 0.8. Inconsistent.
INSUFF

(A)

74. In a given square T we inscribe a circle S. Into that circle we inscribe a square R. Is the
circumference of S greater than 10?
1). The side of square R is greater than 2
2). The side of square T is greater then 4

ANS: (B)
Stmt 1) The side of square R is greater than 2 -> Lets assume that the side of square is R is 2 =>
the radius of the circle S = sqrt(1+1) = sqrt(2). So the circumference of S = 2*pi*sqrt(2) = 8.8.
Since the side of square R could be anything greater than 2, the circumference of the circle
(accordingly) could be anything greater than 8.8. So it could be either less than 10 or greater.
INSUFF

Stmt 2) The side of the square T is greater then 4 -> Let the side of the square T be 4. The radius
of S would be 2. The circumference S = 2*pi*2 = 4*pi. So the minimum value of the circumference
is greater than 10 => circumference will always be greater than 10. SUFF

75. The greater integer less than -|root3 - 3| is:

ANS: -2

Lets start with the value of -|root3-3|.

-|root3-3| = -|3-root3| = -|1.x| = -1.x (since value of root 3 is around 1.7, 3-root3 would be
slightly greater than 1, we don’t necessarily need to have the exact value).

The question asks the greatest integer less than -1.x. (Greatest) Integer less than -1.x is -2.

76. Each of the 45 books is either British or Spanish, either paperback or hardback. If a book is
selected at random, is the probability that a paperback Spanish book will be selected greater than
1/2?

1). 30 books are paperback book.

2). 15 books are Spanish book.

ANS: (B)

Stmt 1) 30 books are paperback books. Among these any number could be Spanish (< = > 22). So
the probability could be anything. INSUFF

Stmt 2) 15 books are Spanish books. The maximum number of of Spanish paper-backs we can have
is when all Spanish books are paperbacks. In this case the probability would be 15/45=1/3. Since
the maximum probability itself is less than 0.5, the actual will definitely be less than 0.5. So the
answer to the question is a firm NO. SUFF

(B)

77. What is the remainder when n is divided by 10?

1). The tens?digit of 11^n is 4

2). The hundreds?digit of 5^n is 6

ANS: (A)

Stmt 1) The tens digit of 11^n is 4 -> This is possible only in case of n=4,14,24,34 ……. In each
case the remainder when divided by 10 is 4. SUFF

Stmt 2) The 100s digit of 5^n is 6 -> n could be 4, 6 etc. INSUFF

78. If m and n are none negative integers, mn=?


1). 4^m=2^n

2). 3^m=2^n

ANS: Same as 57

79. As the figure shows, the three identical circles are tangent to each other. If the area of the blue
region is 64root3-32pi, what is the radius of the circle

ANS: 8

The question becomes easy when you realize how the area of the shaded figure can be calculated.
Join the centers of the three circles, we get an equilateral triangle with each side as 2*r (r is the
radius).

Area of the shaded region = Area of the equilateral triangle – 3* sectors formed at the three
corners =>

64root3-32pi = sqrt(3)*r^2 – 3*pi/6*r^2 => 32(2sqrt(3)-pi) = r^2(2sqrt(3)-pi) => r^2/2 = 32


=> r=sqrt(64)=8

80. If the area of the region encircled by axis-x, axis-y, and y=8x+a is greater than 1, what is the
range of a?

ANS: |a|>4

Given the equation of the line as y=8x+a. We need to get x intercept and y intercept.

Put x=0 => y intercept is a

Put y=0 => x intercept is –a/8.

The area of the triangle formed by a line and X/Y axes is 1/2 * |x intercept|*|y intercept| (Since the
triangle would be a right angled triangle, the x and y intercept will give the measure of base and
height)

Area = 1/2*a*a/8 = a^2/16>1 (given area is greater than 1) => a^2 > 16

=> either a>4 or a<-4 which can be depicted as |a|>4

81. Of the members in an organization, 30% are female, and 3/8 are married male. Which of the
following is true?

I. The number of the female is greater than the number of the married male

II. The number of the female is greater than the number of the un-married male.

III. The number of the married male is greater the number of the un-married male.

ANS: III Only

Lets calculate the percentages clearly.

Females = 30% => Males =70%.


Married Male = 3/8 = 37.5% => Unmarried male = 32.5%

It is clear from these numbers that only condition III holds true.

82. The surface of a Clock (in a dart game) is evenly divided to eight parts numbered with numbers
1 to 8. If we throw arrows three times, how many ways are possible that the total score is 16?

42, 56, 64?/P>

ANS: 42

The question is basically asking how many possible way can you get 16 when you got three turns
and 0-8 numbers possible (repetitions allowed):

Combinations 0 8 8 - 3!/2! Ways (Arranging 3 numbers with 1 number repeated)

1 7 8 – 3! Ways

2 6 8 – 3!

2 7 7 – 3!/2!

3 5 8 – 3!

3 6 7 – 3!

4 4 8 – 3!/2!

4 5 7 – 3!

4 6 6 – 3!/2!

Cant go beyond 4, since 5 is already considered in arrangements above. Total ways = 42.

___

83. When k^4 is divided by 32, the remainder is 0. Which of the following could be the remainder
when k is divided by 32?

2, 4, 6

ANS: 4

Given, k^4 is divisible by 32. We have to find which of (2,4,6) is a possible remainder when k is
divisible 32.

Lets consider 2 first, lets assume k leaves 2 as remainder when divided by 32 => k=32p+2

k^4=(k^2)^2= ((32p+2)^2)^2) = ((32p)^2 + 2*2*32p + 2^2)^2 = (32p^2)^2 + (2*2*32p)^2


+ (2^2)^2 + 2* (32p^2)* (2*2*32p) + 2*(2*2*32p)* (2^2) + 2*(32p^2)* (2^2).

Except for (2^2)^2, all other terms above have 32 as a factor, hence divisible by 32. So there is
one term which is not divisible by 32, hence k^4 is not divisible by 32. Given in the question that
k^4 is divisible by 32. We have reached a contradiction => 2 is not a possible remainder.
Consider 4, The only term which we are really worried about is (4^2)^2, since as above other
terms will be divisible by 32. (4^2)^2 is divisible by 32. Hence k^4 is divisible by 32 => 4 is
possible remainder.

Consider 6, (6^2)^2 is not divisible by 32 => contradiction as in the case of 2 (as remainder).
Hence 6 cannot be a remainder

84. "I" is a positive number. If it is rounded to integer N, E(I)=(N-I)/I. For an instance, if I=1.5,
then E(1.5)=(2-1.5)/1.5=33.33%. E(1.6)-E(2.5)=?

ANS: 5%

Simple calculation. E(1.6) = (2-1.6)/1.6 = 0.4/1.6 = 0.25 and E(2.5)=(3-2.5)/2.5 = 0.5/2.5 = 0.2

E(1.6)-E(2.5) = 0.05 which is same as 5%

85. A number of 5,000 pots are placed in a 25,000 square yard field. Averagely, each pot occupy
how many square feet? 1 yard = 3 feet.

A 15 b 45

ANS: b 45

Average area per pot = 25000/5000 = 5 square yards = 5*3*3 square feet = 45 sq ft.

86. Is x>1?

1). x^2<x

2). x<rootx

ANS: (D)

Stmt 1) x^2<x is only possible for numbers 0<x<1. So we get a definite NO as the answer for the
asked question. SUFF

Stmt 2) x<root(x) -> x has to be positive for root(x) to be valid. Divide both sides by root(x), we
get root(x)<1 => 0<x<1. So we get a definite NO as the answer for the asked question. SUFF

Anda mungkin juga menyukai